Fundamental Counting Principle Proof (NOT via induction)

Click For Summary
The discussion focuses on proving the Fundamental Counting Principle, which states that if set A has m elements and set B has n elements, then the Cartesian product A X B has mn elements. The original poster seeks clarification on the reference to "drawing little trees" as a method for this proof, specifically looking for a non-inductive approach. They express uncertainty about how to start the proof and inquire about the definition of a set having m elements. The conversation highlights the need for a bijective function to establish the relationship between the sets and their respective counts. Overall, the thread aims to clarify the proof process without relying on induction.
Chicago_Boy1
Messages
6
Reaction score
0
Hello all,

I am going through some sample problems exercises in Paul Sally's Tools of the Trade, and am being asked to prove the Fundamental Counting Principle. That is, If A has m elements and B has n elements, then A X B has mn elements.

Sally goes on to write that "this is simple to prove by drawing little trees or using some other artifice."

Basically, I am not really sure what he means by "drawing little trees." Can someone guide me through how I can actually prove this WITHOUT induction?

Thank you in advance.

P.S. I know that I am supposed to show that I've attempted to solve the problem, but honestly I have no idea where to even start.
 
Physics news on Phys.org
What is your definition of "A has m elements" for a set A? It's probably something like "A is in bijective correspondence with the set [m] = \{0, 1, \dots, m - 1\}" (although knowing Sally, he probably starts counting with 1 instead). So you can rephrase the problem as: If f: A \to [m] is a bijection and g: B \to [n] is a bijection, find a bijection h: A \times B \to [mn]. Does that help?
 
Question: A clock's minute hand has length 4 and its hour hand has length 3. What is the distance between the tips at the moment when it is increasing most rapidly?(Putnam Exam Question) Answer: Making assumption that both the hands moves at constant angular velocities, the answer is ## \sqrt{7} .## But don't you think this assumption is somewhat doubtful and wrong?

Similar threads

  • · Replies 4 ·
Replies
4
Views
3K
  • · Replies 6 ·
Replies
6
Views
2K
  • · Replies 3 ·
Replies
3
Views
6K
Replies
7
Views
3K
Replies
2
Views
2K
  • · Replies 3 ·
Replies
3
Views
2K
  • · Replies 3 ·
Replies
3
Views
3K
  • · Replies 1 ·
Replies
1
Views
2K
Replies
5
Views
3K
Replies
4
Views
3K